Math Pro 數學補給站's Archiver

在遇到困難時要具備有三個自我的能力:
自我激勵、自我轉換、自我調節。

Christina 發表於 2017-7-18 16:09

106木柵高工(第二次)

想請教第5題跟第8題,謝謝^_^

thepiano 發表於 2017-7-18 17:55

回復 1# Christina 的帖子

第 8 題
\(f(x)=a_1sinx+a_2sin2x+\ldots+a_n sinnx\),\(a_i\in R,n\in N\)且\(|\;f(x)|\;\le |\;sinx|\;\),\(\forall x\in R\)證明:\(|\;a_1+2a_2+\ldots+na_n|\;\le 1\)
[解答]
\(\left| {{a}_{1}}+2{{a}_{2}}+\cdots +n{{a}_{n}} \right|=\left| f'\left( 0 \right) \right|=\underset{x\to 0}{\mathop{\lim }}\,\left| \frac{f\left( x \right)-f\left( 0 \right)}{x-0} \right|=\underset{x\to 0}{\mathop{\lim }}\,\left| \frac{f\left( x \right)}{x} \right|\le \underset{x\to 0}{\mathop{\lim }}\,\left| \frac{\sin x}{x} \right|=1\)

BambooLotus 發表於 2017-7-18 17:57

第5題
計算\(\displaystyle \int_0^1 \int_x^1 x^2\sqrt{1+y^4}dydx\)
[解答]
\(\displaystyle \int_0^1 {\int_x^1 {{x^2}\sqrt {1 + {y^4}} dydx}  = \int_0^1 {\int_0^y {{x^2}\sqrt {1 + {y^4}} dxdy} } }  = \int_0^1 {\left( {\frac{1}{3}{x^3}\sqrt {1 + {y^4}} \left| {_0^y} \right.} \right)dy} \)
\(\displaystyle = \frac{1}{3}\int_0^1 {{y^3}\sqrt {1 + {y^4}} dy}  = \frac{1}{3} \times \frac{2}{3} \times \frac{1}{4}{\left( {1 + {y^4}} \right)^{\frac{3}{2}}}\left| {_0^1} \right. = \frac{{\sqrt 2 }}{9} - \frac{1}{{18}} \)

thepiano 發表於 2017-7-18 19:31

回復 4# 袁希睿 的帖子

第 6 題
已知函數\(f\)滿足\(f(0)=0,f'(0)=1\),求\(\displaystyle \lim_{x\to 0}\frac{f(x)}{x}\)。某學生作法如下:
∵\(\displaystyle \frac{f(0)}{0}\)為\(\displaystyle \frac{0}{0}\)不定型,故依羅必達法可得
\(\displaystyle \lim_{x\to 0}\frac{f(x)}{x}=\lim_{x\to 0}\frac{f'(x)}{1}=f'(0)=1\)
請問這樣的做法是否正確?若正確是否能有其他解法?若錯誤請舉一反例說明錯誤之處。
[解答]
反例:f(x) = sinx 時,不能用羅必達,會有循環論證的問題

martinofncku 發表於 2017-7-18 21:43

請問老師 9, 10, 11.

BambooLotus 發表於 2017-7-18 21:49

第9題
某圍棋賽由實力相當的甲、乙、丙三棋手參加,規則如下:甲、乙先開始,然後敗者退出由丙遞補重新再下第二盤;接著敗者再退出,再由另一人遞補重新再賽。依此規則最後連勝2局者獲勝,試問最後甲獲勝的機率。
[解答]
令甲贏一場之後獲勝的機率為\( a \),\(\displaystyle a = \frac{1}{2} + \frac{1}{2}\left( {\frac{1}{2} \times 0 + \frac{1}{2}\left( {\frac{1}{2} \times a + \frac{1}{2} \times 0} \right)} \right) \)
然後再去討論整個機率應該就可以了

第11題
試求所有實數\(x\),使得\(\displaystyle x=\sqrt{x-\frac{1}{x}}+\sqrt{1-\frac{1}{x}}\)。
[解答]
[url]https://math.pro/db/viewthread.php?tid=1492&page=2#pid7156[/url]

tsusy 發表於 2017-7-18 22:47

回復 6# martinofncku 的帖子

10.
一長方體內部對角線到三條與他不相交的稜之間的最短距離分別為\(\displaystyle 2\sqrt{5},\frac{30}{\sqrt{13}},\frac{15}{\sqrt{10}}\),試求此長方體的體積。
[解答]
假設長方體三個不同方向的邊長分別為 \( a, b, c \)

計算三組歪斜距離可得 \(\displaystyle \frac{bc}{\sqrt{b^{2}+c^{2}}}, \frac{ca}{\sqrt{c^{2}+a^{2}}}, \frac{ab}{\sqrt{a^{2}+b^{2}}} \)

因對稱性,不妨設

\( \begin{cases} \displaystyle
\frac{bc}{\sqrt{b^{2}+c^{2}}} & =2\sqrt{5}\\
\frac{ca}{\sqrt{c^{2}+a^{2}}} & =\frac{30}{\sqrt{13}}\\
\frac{ab}{\sqrt{a^{2}+b^{2}}} & =\frac{15}{\sqrt{10}}
\end{cases} \)

由第一式、第三式可得 \( c^{2}=\frac{20b^{2}}{b^{2}-20}, a^{2}=\frac{45b^{2}}{2b^{2}-45} \)

代入第二式得 \( \frac{20b^{2}}{b^{2}-20}\cdot\frac{45b^{2}}{2b^{2}-45}=\frac{900}{13}\left(\frac{20b^{2}}{b^{2}-20}+\frac{45b^{2}}{2b^{2}-45}\right) \)

左右同乘 \( \frac{13}{900b^2} (b^{2}-20)(2b^{2}-45) \),化簡得

\( 13b^{2}=85b^{2}-1800 \)

故 \( b=5, c=10, a=15 \),體積為 750

cefepime 發表於 2017-7-18 23:58

9.
某圍棋賽由實力相當的甲、乙、丙三棋手參加,規則如下:甲、乙先開始,然後敗者退出由丙遞補重新再下第二盤;接著敗者再退出,再由另一人遞補重新再賽。依此規則最後連勝2局者獲勝,試問最後甲獲勝的機率。
[解答]
[size=3]想到兩個本質上相同的方法[/size]
[size=3][b]方法一[/b][/size]
[size=3]令 P(贏),P(輸),與 P(補) 依序表示賽程中,剛贏一局者,剛輸一局者,與遞補者最後獲勝的機率。[/size]
[size=3][/size]
[size=3]則: P(補) = P(贏) /2  ;  P(輸) = P(補) /2[/size]
[size=3][/size]
[size=3]⇒ P(贏) = 4/7,P(補) = 2/7,P(輸) = 1/7[/size]
[size=3][/size]
[size=3]所求 = (1/2)*[ P(贏) + P(輸) ] = 5/14[/size]
[size=3][/size]
[size=3][/size]
[size=3][b]方法二[/b][/size]
[size=3][/size]
[size=3]令所求 = p,則丙最後獲勝的機率 = 1-2p。[/size]
[size=3][/size]
[size=3]則一局後,甲乙的負方,勝方,與丙,最後獲勝的機率依序為 (1-2p)/2 ; (1/2)+(1-2p)/4 ; 1-2p。[/size]
[size=3][/size]
[size=3]由三者之和 = 1,得 p = 5/14[/size]

cut6997 發表於 2017-7-21 03:25

各位老師我想要請教一下第二題
原本嘗試過加總回頭消和同除獲同除abc都不見效果
後來有人建議用兩式相減成功提出題目要求的a+b+c的因式
但之後我卻解不出答案
似乎在相減的過程中把條件給刪去了
我把相減後的結果丟給wolffram 一樣也只能求出比例關係
不知道是哪裡出了問題
以下是原式丟給wolffram 和兩兩相減丟給wolffram的連結
[url=http://www.wolframalpha.com/input/?source=frontpage-immediate-access&i=solve%5B%7Ba%5E2%2Bb%5E2%2Ba*b%3D9%7D,%7Bb%5E2%2Bc%5E2%2Bb*c%3D16%7D,%7Bc%5E2%2Ba%5E2%2Bc*a%3D25%7D,%7Bt%3Da%2Bb%2Bc%7D,%7Ba%3E0%7D,%7Bb%3E0%7D,%7Bc%3E0%7D%5D]http://www.wolframalpha.com/input/?source=frontpage-immediate-access&i=solve%5B%7Ba%5E2%2Bb%5E2%2Ba*b%3D9%7D,%7Bb%5E2%2Bc%5E2%2Bb*c%3D16%7D,%7Bc%5E2%2Ba%5E2%2Bc*a%3D25%7D,%7Bt%3Da%2Bb%2Bc%7D,%7Ba%3E0%7D,%7Bb%3E0%7D,%7Bc%3E0%7D%5D[/url]

[url=http://www.wolframalpha.com/input/?source=frontpage-immediate-access&i=solve%5B%7B(c-a)*t%3D7%7D,%7B(a-b)*t%3D9%7D,%7B(c-b)*t%3D16%7D,%7Bt%3Da%2Bb%2Bc%7D,%7Ba%3E0%7D,%7Bb%3E0%7D,%7Bc%3E0%7D%5D]http://www.wolframalpha.com/input/?source=frontpage-immediate-access&i=solve%5B%7B(c-a)*t%3D7%7D,%7B(a-b)*t%3D9%7D,%7B(c-b)*t%3D16%7D,%7Bt%3Da%2Bb%2Bc%7D,%7Ba%3E0%7D,%7Bb%3E0%7D,%7Bc%3E0%7D%5D[/url]

bugmens 發表於 2017-7-21 03:57

2.
已知\( a,b,c \)為正實數,\( \displaystyle \Bigg\{\ \matrix{a^2+b^2+ab=9 \cr b^2+c^2+bc=16 \cr c^2+a^2+ca=25 } \),求\( a+b+c= \)?
解答
[url]https://math.pro/db/viewthread.php?tid=937&page=1#pid2033[/url]


11.
試求所有實數\(x\),使得\( \displaystyle x=\sqrt{x-\frac{1}{x}}+\sqrt{1-\frac{1}{x}} \)
解答
[url]https://math.pro/db/viewthread.php?tid=1492&page=2#pid7156[/url]

goodluck 發表於 2017-7-27 12:56

回復 5# thepiano 的帖子

請問老師,這題的f ' (0)=1 是已知條件
這樣還會有循環論證的問題嗎?

thepiano 發表於 2017-7-27 16:09

回復 12# goodluck 的帖子

在證明\({{\left( \sin x \right)}^{'}}=\cos x\)時,會用到\(\underset{x\to 0}{\mathop{\lim }}\,\frac{\sin x}{x}=1\)
而\(\cos 0=1\),故有循環論證的問題

goodluck 發表於 2017-7-28 10:25

回復 13# thepiano 的帖子

老師您好
我的意思是,這題的 f ' (0)=1是已知條件
所以不需要證明 f ' (x)是什麼函數,
題目已知f ' (0)=1
那這還會有循環論證的問題嗎?

laylay 發表於 2017-7-28 11:38

1.

已知有理數\(\displaystyle k=\frac{11\times26+12\times27+13\times28+14\times29+15\times30}{11\times25+12\times26+13\times27+14\times28+15\times29}\),求\(100k\)的整數部分。
[解答]
令n=13
則 k=1+n/(2n^2+n+2)=1+1/(2n+1+2/n)=1+1/27.1...=1.036....
[100k]=103
[1000k]=1036

cefepime 發表於 2017-7-28 13:22

1.
已知有理數\(\displaystyle k=\frac{11\times26+12\times27+13\times28+14\times29+15\times30}{11\times25+12\times26+13\times27+14\times28+15\times29}\),求\(100k\)的整數部分。
[另解]
[size=3]觀察到 k 是由 26/25,27/26,28/27,29/28,30/29 這五個值,分子分母分別相加後所得。[/size]
[size=3][/size]
[size=3]故: 26/25 > k > 30/29  (我覺得可視之為 "糖水不等式" 的推廣; 或用代數證明亦不難)[/size]
[size=3][/size]
[size=3]⇒ 104 > 100k > 103[b].∙∙∙[/b][/size]
[size=3][/size]
[size=3]⇒ 所求 = 103[/size]
[size=3][/size]
[size=3][/size]

laylay 發表於 2017-7-28 13:22

4.

求所有的整數\(k\),使得\(6k^2-7k-5\)為某個質數的平方。
[解答]
原式=(3k-5)(2k+1)=P^2
=> 3k-5, 2k+1=+-1 都不合
或 3k-5=2k+1=> k=6 , P=13(合)

laylay 發表於 2017-7-28 14:08

3.

求多項式\(f(x)=x^{2006}-x^{2004}-x^{2002}-\ldots-x^2-2\)的所有實根的平方和。
[解答]
f(x)=(x^2006-1)-(x^2004+x^2002+....+x^2+1)=0
=>f(x)(x^2-1)=(x^2006-1)(x^2-1)-(x^2006-1)=(x^2006-1)(x^2-2)=0
=>x=1,-1,ㄏ2,-ㄏ2
但f(x)=0 沒有1,-1的實根
可知f(x)=0 只有ㄏ2,-ㄏ2的實根
=>所求=2+2=4

laylay 發表於 2017-7-28 14:43

7.

平面上有一個面積為1的凸四邊形\(ABCD\),已知\(\overline{AB}// \overline{DC}\),且對角線\(\overline{AC},\overline{BD}\)交於\(O\)點,求\(\Delta AOD\)面積的最大值。
[解答]
COD與AOB為相似形
設CO=rAO,AOB的面積為a
則AOD的面積=BOC的面積=ra,DOC的面積=rra
=>a(1+r+r+rr)=1=>AOD的面積=ra=r/(1+r+r+rr)=1/(r+1/r+2)<=1/4為所求

laylay 發表於 2017-7-28 17:10

12.

\(\Delta ABC\)中,\(\overline{AB}=4\),\(\overline{BC}=5\),\(\overline{CA}=7\),\(P\)為任意一點,試求\(\overline{PA}^2+\overline{PB}^2+\overline{PC}^2\)的最小值。
[解答]
設P(x,y),A(x1,y1)......可知P為重心
=>所求=(4^2+5^2+7^2)/3=30

thepiano 發表於 2017-7-28 20:19

回復 14# goodluck 的帖子

您好:

小弟只是舉一個符合題目條件的\(f\left( x \right)\),但在求\(\underset{x\to 0}{\mathop{\lim }}\,\frac{f\left( x \right)}{x}\)時,不適用羅必達法則的例子。

如果您覺得沒有循環論證的問題,那就沒有吧!

頁: [1] 2

論壇程式使用 Discuz! Archiver   © 2001-2022 Comsenz Inc.